Đến nội dung

Hình ảnh

Topic tổng hợp một số bất đẳng thức trong kì thi MO các nước

* * * * * 16 Bình chọn

  • Please log in to reply
Chủ đề này có 501 trả lời

#381
dogsteven

dogsteven

    Đại úy

  • Thành viên
  • 1567 Bài viết

Dám hỏi dấu bằng xảy ra khi nào???

$a=b=1, c=0$ và các hoán vị.


Quyết tâm off dài dài cày hình, số, tổ, rời rạc.


#382
dangkhuong

dangkhuong

    Sĩ quan

  • Thành viên
  • 312 Bài viết

Bài 139(Kazakstan MO): Cho a,b,c>0. CMR:

$\sum \frac{1}{a^2+ab+b^2}\geq \frac{9}{(a+b+c)^2}$


:ukliam2:  :ukliam2:  :ukliam2:


#383
Belphegor Varia

Belphegor Varia

    Thượng sĩ

  • Thành viên
  • 227 Bài viết

Bài 121 (CĐTMO 1993) : Xét các số thực $x_{1},x_{2},x_{3},x_{4}$ thỏa mãn 
                                                      $\frac{1}{2}\leq x_{1}^{2}+x_{2}^{2}+x_{3}^{2}+x_4^2\leq 1.$

                                        

                                         Tìm giá trị lớn nhất và nhỏ nhất của biểu thức 
                                       $A=(x_1-2x_2+x_3)^2+(x_2-2x_3+x_4)^2+(x_2-2x_1)^2+(x_3-2x_4)^2$

Bài 122 (CĐTMO 1991) : Cho dãy số thực dương $a_1,a_2,...,a_n (n>2)$ $;a_1\neq a_n$ , là dãy không giảm , hoặc là dãy không tăng ,

                                        và cho các số thực dương $x,y$ thoả mãn điều kiện $\frac{x}{y}\geq \frac{a_1-a_2}{a_1-a_n}.$ 

                                        Chứng minh rằng 

                                       $\frac{a_1}{a_2x+a_3y}+...+\frac{a_k}{a_{k+1}x+a_{k+2}y}+...+\frac{a_{n-2}}{a_{n-1}x+a_ny}+\frac{a_{n-1}}{a_nx+a_1y}+\frac{a_n}                                                    {a_1x+a_2y}\geq \frac{n}{x+y}$

Còn sót 2 bài chưa có lời giải 


$ \textbf{NMQ}$

Wait a minute, You have enough time. Also tomorrow will come 

Just take off her or give me a ride 

Give me one day or one hour or just one minute for a short word 

 


#384
ducvipdh12

ducvipdh12

    Sĩ quan

  • Thành viên
  • 454 Bài viết

Bài 140 ( Peru TST 2007 ):

Cho  $a_1,a_2,...,a_n\in \left [ 0,1 \right ];S=a_1^3+a_2^3+...+a_n^3. CMR:$

$\frac{a_1}{2n+1+S-a_1^3}+\frac{a_2}{2n+1+S-a_2^3}+...+\frac{a_n}{2n+1+S-a_n^3}\leq \frac{1}{3}$

Lâu lâu post cái đề :D


FAN THẦY THÔNG,ANH CẨN,THẦY VINH :icon6: :icon6:

#385
Nguyen Minh Hai

Nguyen Minh Hai

    Thiếu úy

  • Thành viên
  • 666 Bài viết

Bài 139(Kazakstan MO): Cho a,b,c>0. CMR:

$\sum \frac{1}{a^2+ab+b^2}\geq \frac{9}{(a+b+c)^2}$

BĐT cần chứng minh tương đương với

$\sum \frac{a^2+b^2+c^2+ab+bc+ca}{a^2+ab+b^2} \geq \frac{9(a^2+b^2+c^2+ab+bc+ca)}{(a+b+c)^2}$

 

$\Leftrightarrow \sum \left ( \frac{(a^2+ab+b^2)+c(a+b+c)}{a^2+b^2+c^2} \right ) \geq \frac{9(a^2+b^2+c^2+ab+bc+ca)}{(a+b+c)^2}$

 

$\Leftrightarrow 3+(a+b+c).\sum \frac{c}{a^2+ab+b^2} \geq \frac{9(a^2+b^2+c^2+ab+bc+ca)}{(a+b+c)^2}$

 

Áp dụng $Cauchy-Schwarz$ ta có:

$\sum \frac{c}{a^2+ab+b^2}\geq \frac{(a+b+c)^2}{\sum c(a^2+ab+b^2)}=\frac{a+b+c}{ab+bc+ca}$

 

Do đó ta cần chứng minh:

$3+\frac{(a+b+c)^2}{ab+bc+ca} \geq \frac{9(a^2+b^2+c^2+ab+bc+ca)}{(a+b+c)^2}$

 

$\Leftrightarrow 3+\frac{(a+b+c)^2}{ab+bc+ca}\geq \frac{9[(a+b+c)^2-(ab+bc+ca)]}{(a+b+c)^2}$

 

$\Leftrightarrow \frac{(a+b+c)^2}{ab+bc+ca}+\frac{9(ab+bc+ca)}{(a+b+c)^2} \geq 6$

 

Tuy nhiên BĐT cuối luôn đúng theo $AM-GM$

Do đó bài toán được chứng minh xong!



#386
olympiachapcanhuocmo

olympiachapcanhuocmo

    Thượng sĩ

  • Thành viên
  • 208 Bài viết

Tớ có cách khác :$\sum \frac{c^{2}}{a^{2}c^{2}+abc^{2}+b^{2}c^{2}}\geq \sum \frac{\left ( a+b+c \right )^{2}}{2\sum a^{2}c^{2}+abc\left ( a+b+c \right )}\geq (\frac{a+b+c}{ab+bc+ca})^{2}\geq \left ( \frac{3}{a+b+c} \right )^{2}=\frac{9}{\left ( a+b+c \right )^{2}}$


                                                                                               


#387
dangkhuong

dangkhuong

    Sĩ quan

  • Thành viên
  • 312 Bài viết

Bài 141(Pakistan TST): Cho a,b,c>0. CMR:nếu ab+bc+ca=3 thì

$\sum \frac{1}{a^2+b^2(c^2+1)}\leq \frac{a^2+b^2+c^2}{3}$

 


:ukliam2:  :ukliam2:  :ukliam2:


#388
marcoreus101

marcoreus101

    Thượng sĩ

  • Thành viên
  • 235 Bài viết

Không biết bài này có chưa, nếu có rồi thì anh Đức xóa dùm em

Bài 142(APMO 1996) Cho a,b,c là độ dài 3 cạnh tam giác. Chứng minh rằng:

$\sqrt{a+b-c}+\sqrt{b+c-}+\sqrt{c+a-b}\leq \sqrt{a}+\sqrt{b}+\sqrt{c}$

Đẳng thức xảy ra khi nào


Bài viết đã được chỉnh sửa nội dung bởi marcoreus101: 23-06-2015 - 15:16


#389
dangkhuong

dangkhuong

    Sĩ quan

  • Thành viên
  • 312 Bài viết

Không biết bài này có chưa, nếu có rồi thì anh Đức xóa dùm em

Bài 142(APMO 1996) Cho a,b,c là độ dài 3 cạnh tam giác. Chứng minh rằng:

$\sqrt{a+b-c}+\sqrt{b+c-}+\sqrt{c+a-b}\geq \sqrt{a}+\sqrt{b}+\sqrt{c}$

Đẳng thức xảy ra khi nào

Hình như bị ngược dấu rồi. Bạn xem lại dùm


:ukliam2:  :ukliam2:  :ukliam2:


#390
dangkhuong

dangkhuong

    Sĩ quan

  • Thành viên
  • 312 Bài viết

Bài 137(Brirstish MO): Cho a,b,c>0. CMR: nếu a+b+c=3.

$\frac{a}{b^2+8bc+3c^2}+\frac{b}{c^2+8ca+3a^2}+\frac{c}{a^2+8ab+3b^2}\geq \frac{1}{4}$

Sau đây là lời giải cho bài toán trên:

Ta giả sử a$\geq$b$\geq$c

Khi đó dễ thấy rằng:$a\geq b\geq c$ $\frac{1}{b^2+8bc+3c^2}\geq \frac{1}{c^2+8ca+3a^2}\geq \frac{1}{a^2+8ab+3b^2}$

Áp dụng bất đẳng thức Chebysev và bất đẳng thức AM-GM, ta có:

$\sum \frac{a}{b^2+4bc+3c^2}\geq \frac{1}{3}(a+b+c)(\sum \frac{1}{b^2+4bc+3c^2})\geq 3(a+b+c)(\frac{1}{4(a+b+c)^2})\geq \frac{1}{4}$ (đpcm)


Bài viết đã được chỉnh sửa nội dung bởi dangkhuong: 23-06-2015 - 17:05

:ukliam2:  :ukliam2:  :ukliam2:


#391
Nguyen Minh Hai

Nguyen Minh Hai

    Thiếu úy

  • Thành viên
  • 666 Bài viết

Không biết bài này có chưa, nếu có rồi thì anh Đức xóa dùm em

Bài 142(APMO 1996) Cho a,b,c là độ dài 3 cạnh tam giác. Chứng minh rằng:

$\sqrt{a+b-c}+\sqrt{b+c-}+\sqrt{c+a-b}\geq \sqrt{a}+\sqrt{b}+\sqrt{c}$

Đẳng thức xảy ra khi nào

Đề sai rồi. Đề đúng phải là: $\sqrt{a+b-c}+\sqrt{b+c-a}+\sqrt{c+a-b} \leq \sqrt{a}+\sqrt{b}+\sqrt{c}$

Ta đặt: $\left\{\begin{matrix} a+b-c=z & & & \\ b+c-z=x & & & \\ c+a-b =y& & & \end{matrix}\right.$

$\Rightarrow a=\frac{y+z}{2};b=\frac{z+x}{2};c=\frac{x+y}{2}$

 

BĐT cần chứng minh trở thành: $\sqrt{2}(\sqrt{x}+\sqrt{y}+\sqrt{z}) \leq \sqrt{x+y}+\sqrt{y+z}+\sqrt{z+x}$

$\Leftrightarrow 2(\sqrt{x}+\sqrt{y}+\sqrt{z}) \leq \sqrt{2(x+y)}+\sqrt{2(y+z)}+\sqrt{2(x+z)}$

 

Áp dụng BĐT $Buniakowski$ ta có:

$\left\{\begin{matrix} \sqrt{2(x+y)} \geq \sqrt{x}+\sqrt{y} & & & \\ \sqrt{2(y+z)} \geq \sqrt{y}+\sqrt{z}& & & \\ \sqrt{2(z+x)} \geq \sqrt{z}+\sqrt{x} & & & \end{matrix}\right.$

 

Do đó ta có đpcm.



#392
an1712

an1712

    Trung sĩ

  • Thành viên
  • 149 Bài viết

Câu 13 :(Balkan MO , 2014) Cho $x,y,z>0$ thỏa : $xy+yz+zx=3xyz$ . Chứng minh rằng : 

$x^{2}y+y^{2}z+z^{2}x\geq 2(x+y+z)-3$

 

P/s : Mong các bạn tham gia sôi nổi nhé ! :biggrin:

áp dụng holder:

$(x^2y+y^2z+z^2x)(\sum \frac{1}{x})\geq (x+y+z)^2$ 

$\sum x^2y\geq \frac{xyz(x+y+z)^2}{xy+yz+xz}\doteq \frac{(x+y+z)^2}{3}$

áp dụng cố si:

$\frac{(x+y+z)^2}{3}+3\geq 2(x+y+z) (đpcm)$


tiến tới thành công  :D


#393
binhnhaukhong

binhnhaukhong

    Sĩ quan

  • Thành viên
  • 343 Bài viết

Bài 141(Pakistan TST): Cho a,b,c>0. CMR:nếu ab+bc+ca=3 thì

$\sum \frac{1}{a^2+b^2(c^2+1)}\leq \frac{a^2+b^2+c^2}{3}$

 

Ý tưởng là đơn giản được tối đa biểu thức,tránh những đánh giá phức tạp dẫn đến lời giải dài dòng ( vừa làm theo $p,q,r$).

Ở đây dùng $C-S$ để đánh giá là dễ nghĩ đến. Vì vậy ta sẽ ghép cặp sao cho sau khi đánh giá các phân thức đều có chung mẫu thức sẽ dễ dàng hơn cho các biến đổi sau:

 

$(a^2+b^2c^2+c^2)(b^2+1+a^2)\geq (ab+bc+ac)^2=9$

 

Rất may mắn cho ta là đánh giá này đồng thời làm xuất hiện những đại lượng mà ta mong muốn ở bên vế phải,vì vậy ta tự tin là đánh giá này sẽ đem đến kết quả.

 

Thật vậy làm tương tự với các phân thức sau thì BĐT của ta quy về CM BĐT đơn giản sau:

 

$a^2+b^2+c^2\geq 3=ab+bc+ac$


Quy Ẩn Giang Hồ. 

So goodbye!

 

:off:  :off:  :off:  :off:  :off:  :off: 


#394
dogsteven

dogsteven

    Đại úy

  • Thành viên
  • 1567 Bài viết

Sau đây là lời giải cho bài toán trên:

Không mất tính tổng quát ta giả sử a$\geq$b$\geq$c

Khi đó dễ thấy rằng:$a\geq b\geq c$ $\frac{1}{b^2+8bc+3c^2}\geq \frac{1}{c^2+8ca+3a^2}\geq \frac{1}{a^2+8ab+3b^2}$

Áp dụng bất đẳng thức Chebysev và bất đẳng thức AM-GM, ta có:

$\sum \frac{a}{b^2+4bc+3c^2}\geq \frac{1}{3}(a+b+c)(\sum \frac{1}{b^2+4bc+3c^2})\geq 3(a+b+c)(\frac{1}{4(a+b+c)^2})\geq \frac{1}{4}$ (đpcm)

Nên xem lại.


Quyết tâm off dài dài cày hình, số, tổ, rời rạc.


#395
dogsteven

dogsteven

    Đại úy

  • Thành viên
  • 1567 Bài viết

Không biết bài này có chưa, nếu có rồi thì anh Đức xóa dùm em

Bài 142(APMO 1996) Cho a,b,c là độ dài 3 cạnh tam giác. Chứng minh rằng:

$\sqrt{a+b-c}+\sqrt{b+c-}+\sqrt{c+a-b}\geq \sqrt{a}+\sqrt{b}+\sqrt{c}$

Đẳng thức xảy ra khi nào

Bị ngược dấu rồi em.

Không mất tính tổng quát, giả sử $a\geslant b\geqslant c$

Dễ thấy $a+b-c\geqslant c+a-b\geqslant b+c-a$

Mà $a+b-c\geqslant a$ và $a+b-c+c+a-b=2a\geqslant a+b$ và $a+b-c+c+a-b+b+c-a=a+b+c$ nên $(a+b-c, c+a-b, b+c-a)\succ (a,b,c)$

Áp dụng bất đẳng thức Karamata cho hàm lõm $f(x)=\sqrt{x}$ cho ta điều phải chứng minh.


Quyết tâm off dài dài cày hình, số, tổ, rời rạc.


#396
an1712

an1712

    Trung sĩ

  • Thành viên
  • 149 Bài viết

 

$(a^2+b^2c^2+c^2)(b^2+1+a^2)\geq (ab+bc+ac)^2=9$

 

cái đó hình như bị nhầm rồi biểu thức là $a^2+b^2(c^2+1)=a^2+b^2c^2+b^2$


tiến tới thành công  :D


#397
dangkhuong

dangkhuong

    Sĩ quan

  • Thành viên
  • 312 Bài viết

Bài 143(APMO Shortlist): Cho a,b,c>0. CMR: nếu ab+bc+ca=1

$\frac{1}{a^2(b+c)+bc^2}+\frac{1}{b^2(c+a)+ca^2}+\frac{1}{c^2(a+b)+ab^2}\leq \frac{1}{a}+\frac{1}{b}+\frac{1}{c}$

 


:ukliam2:  :ukliam2:  :ukliam2:


#398
khanghaxuan

khanghaxuan

    Trung úy

  • Thành viên
  • 969 Bài viết

Lời giải bài 143 :

Để ý cái mẫu $a^{2}(b+c)+bc^{2}=a^{2}b+a^{2}c+bc^{2}$ và cái điều kiện $ab+bc+ca=1$ thì không gì dễ hơn C-S : 

$(a^{2}b+a^{2}c+bc^{2})(b+c+b)\geq (ab+bc+ca)^{2}=1\Rightarrow \frac{1}{a^{2}b+a^{2}c+bc^{2}}\leq 2b+c$

Do đó ta cần chứng minh : $3(a+b+c)\leq \frac{1}{a}+\frac{1}{b}+\frac{1}{c}\Leftrightarrow 3abc(a+b+c)\leq ab+bc+ca\Leftrightarrow (ab+bc+ca)^{2}\geq 3abc(a+b+c)$ (đúng :)) )

 

Spoiler


Bài viết đã được chỉnh sửa nội dung bởi khanghaxuan: 23-06-2015 - 15:38

Điều tôi muốn biết trước tiên không phải là bạn đã thất bại ra sao mà là bạn đã chấp nhận nó như thế nào .

- A.Lincoln -

#399
dangkhuong

dangkhuong

    Sĩ quan

  • Thành viên
  • 312 Bài viết

Bài 144(Poland MO): Cho a,b,c>0. CMR: nếu ab+bc+ca=1 thì

$\frac{(a^2+1)^2}{bc(b+c)}+\frac{(b^2+1)^2}{ca(c+a)}+\frac{(c^2+1)^2}{ab(a+b)}\geq 8(\sqrt{ab}+\sqrt{bc}+\sqrt{ca})$


:ukliam2:  :ukliam2:  :ukliam2:


#400
dangkhuong

dangkhuong

    Sĩ quan

  • Thành viên
  • 312 Bài viết

Cũng chả ảnh hưởng nhiều cách làm vẫn thế

Khổ quá xin hai bạn đừng cãi nhau nữa :(  :( . Vẫn là cách sử dụng Bđt C-S thôi có điều đánh giá như sau:

$(a^2+1+b^2)(b^2+c^2a^2+c^2)\geq (ab+bc+ca)^2=9$.

Từ đó tương tự với hai hạng tử còn lại kết hợp vs bđt AM-GM suy ra đpcm.


:ukliam2:  :ukliam2:  :ukliam2:





1 người đang xem chủ đề

0 thành viên, 1 khách, 0 thành viên ẩn danh